Question and Answers Forum

All Questions      Topic List

Limits Questions

Previous in All Question      Next in All Question      

Previous in Limits      Next in Limits      

Question Number 26764 by abdo imad last updated on 29/Dec/17

find  lim_(n−>∝)  (1/n) ln( Π_(k=1) ^(n−1)  (1− (k/n))).

$${find}\:\:{lim}_{{n}−>\propto} \:\frac{\mathrm{1}}{{n}}\:{ln}\left(\:\prod_{{k}=\mathrm{1}} ^{{n}−\mathrm{1}} \:\left(\mathrm{1}−\:\frac{{k}}{{n}}\right)\right). \\ $$

Commented byprakash jain last updated on 29/Dec/17

Π_(k=1) ^(n−1) (((n−1)!)/n^(n−1) )  lim_(n→∞) ((ln (((n−1)!)/n^(n−1) ))/n)  =lim_(n→∞) ((ln (n−1)!−(n−1)ln n)/n)  will continue later

$$\underset{{k}=\mathrm{1}} {\overset{{n}−\mathrm{1}} {\prod}}\frac{\left({n}−\mathrm{1}\right)!}{{n}^{{n}−\mathrm{1}} } \\ $$ $$\underset{{n}\rightarrow\infty} {\mathrm{lim}}\frac{\mathrm{ln}\:\frac{\left({n}−\mathrm{1}\right)!}{{n}^{{n}−\mathrm{1}} }}{{n}} \\ $$ $$=\underset{{n}\rightarrow\infty} {\mathrm{lim}}\frac{\mathrm{ln}\:\left({n}−\mathrm{1}\right)!−\left({n}−\mathrm{1}\right)\mathrm{ln}\:{n}}{{n}} \\ $$ $$\mathrm{will}\:\mathrm{continue}\:\mathrm{later} \\ $$

Commented byabdo imad last updated on 30/Dec/17

let remember that if f is Rieman integrable on [a,b]  ∫_a ^b f(x)dx = lim_(n−>∝)   ((b−a)/n)Σ_(k=0) ^(n−1) f(a +k((b−a)/n) )  so  lim_(n−>∝) (1/n) ln( Π_(k=1) ^(k=n) ( 1− (k/n)))=lim_(n−>∝) ((1−0)/n) Σ_(k=1) ^n ln(1−k((1−0)/n))  = ∫_0 ^1 ln(1−x)dx =_(1−x=t) −∫_1 ^0 ln(t)dt = ∫_0 ^1 ln(t)dt   =[tln(t)−t]_0 ^1  = −1.

$${let}\:{remember}\:{that}\:{if}\:{f}\:{is}\:{Rieman}\:{integrable}\:{on}\:\left[{a},{b}\right] \\ $$ $$\int_{{a}} ^{{b}} {f}\left({x}\right){dx}\:=\:{lim}_{{n}−>\propto} \:\:\frac{{b}−{a}}{{n}}\sum_{{k}=\mathrm{0}} ^{{n}−\mathrm{1}} {f}\left({a}\:+{k}\frac{{b}−{a}}{{n}}\:\right)\:\:{so} \\ $$ $${lim}_{{n}−>\propto} \frac{\mathrm{1}}{{n}}\:{ln}\left(\:\prod_{{k}=\mathrm{1}} ^{{k}={n}} \left(\:\mathrm{1}−\:\frac{{k}}{{n}}\right)\right)={lim}_{{n}−>\propto} \frac{\mathrm{1}−\mathrm{0}}{{n}}\:\sum_{{k}=\mathrm{1}} ^{{n}} {ln}\left(\mathrm{1}−{k}\frac{\mathrm{1}−\mathrm{0}}{{n}}\right) \\ $$ $$=\:\int_{\mathrm{0}} ^{\mathrm{1}} {ln}\left(\mathrm{1}−{x}\right){dx}\:=_{\mathrm{1}−{x}={t}} −\int_{\mathrm{1}} ^{\mathrm{0}} {ln}\left({t}\right){dt}\:=\:\int_{\mathrm{0}} ^{\mathrm{1}} {ln}\left({t}\right){dt}\: \\ $$ $$=\left[{tln}\left({t}\right)−{t}\right]_{\mathrm{0}} ^{\mathrm{1}} \:=\:−\mathrm{1}. \\ $$

Commented byprakash jain last updated on 31/Dec/17

Thanks understood.  But any other suggestion like  some formula for n!?

$$\mathrm{Thanks}\:\mathrm{understood}. \\ $$ $$\mathrm{But}\:\mathrm{any}\:\mathrm{other}\:\mathrm{suggestion}\:\mathrm{like} \\ $$ $$\mathrm{some}\:\mathrm{formula}\:\mathrm{for}\:{n}!? \\ $$

Terms of Service

Privacy Policy

Contact: info@tinkutara.com